Consider the probability model with sample space (A,B,C} and P(A)=0.1, P(B)=0.1, P(C)=0.8.Then

Consider The Probability Model With Sample Space (A,B,C} And P(A)=0.1, P(B)=0.1, P(C)=0.8.Then

Answers

Answer 1

The sum of all the possible events is 1, which means that all the events are independent of the others, in that case

P(A or C) = P(A)+P(C) = 0.1 + 0.8 = 0.9

P(B and C) = P(B)*P(C) = 0.1 * 0.8 = 0.08


Related Questions

Determine g(x + a) - g(x) for the following function.g(x) = 3x^2 - 5x

Answers

Answer:

3a² + 6ax + 5a

Explanation:

Given the following function

g(x) = 3x² - 5x

Determine g(x+a)

g(x+a) = 3(x+a)² - 5(x+a)

Expand the result

g(x+a) = 3(x² + 2ax + a²) - (5x - 5a)

g(x+a) = 3x² + 6ax + 3a² - 5x + 5a

Determine g(x + a) - g(x)

g(x + a) - g(x) = 3x² + 6ax + 3a² - 5x + 5a - (3x² - 5x)

g(x + a) - g(x) = 3x² + 6ax + 3a² - 5x + 5a - 3x² + 5x

Collect the like terms

g(x + a) - g(x) = 3x² - 3x² - 5x + 5x + 6ax + 3a² + + 5a

g(x + a) - g(x) = 6ax + 3a² + 5a

g(x + a) - g(x) = 3a² + 6ax + 5a

Hence the required result for the function g(x + a) - g(x) is 3a² + 6ax + 5a

PLEASE HELP ME WHAT IS. THIS X^2+5x+4

Answers

Answer: (x+1)(x+4) is the solution

Step-by-step explanation:

Use the sum-product pattern

Common factor from the two pairs

Rewrite in factored form

Answer:

(x+1)(x+4)

Step-by-step explanation:

x²+5x+4

x²+4x+x+4

x(x+4)+1(x+4)

(x+1)(x+4)

[tex]\frac{(438)^{3} \sqrt{0.056} }{(388)^{4} }[/tex]

Answers

The simplification of the given expression is 0.3404

What is simplification of an expression?

Usually, simplification involves proceeding with the pending operations in the expression. Like, 5 + 2 is an expression whose simplified form can be obtained by doing the pending addition, which results in 7 as its simplified form. Simplification usually involves making the expression simple and easy to use later.

WE have given the expression as;

[tex]\frac{(438)^{3} \sqrt{0.056} }{(388)^{4} }[/tex]

Now solving;

[tex]\dfrac{(438)^{3} \sqrt{0.056} }{(388)^{4} }\\\\\\\dfrac{84027784 \times 0.23}{58411072 }\\[/tex]

= 0.3404

Therefore, the simplification of the given expression is 0.3404

Learn more about simplification;

https://brainly.com/question/17579585

#SPJ1

This was the answer I got but I am not sure it it is correct.

Answers

To find JL

We will first find the value of x

JK = KL

2(x+6) = 3x + 8

2x + 12 = 3x+ 8

subtract 2x from both-side of the equation

12 = 3x-2x + 8

12 = x+ 8

subtract 8 from both-side of the equation

12 - 8 = x+ 8 - 8

4 = x

x=4

JK = 2(x+ 6)

substitute x = 4 into the above

JK = 2(4+6)

JK=2(10)

JK=20

KL =3x+8

substitute x=4 into the above

KL = 3(4) + 8

KL = 12 + 8

KL=20

JL = JK + KL

= 20 + 20

= 40

JL = 40 units

The volume of a box is 0.001 cubic Units. If the box is a cube, what is the length of one of its edges?

Answers

Answer:

0.1 units

Step-by-step explanation:

The volume formula of a cube is V = s^3
0.001 = s^3

s = 0.1

Use the graph to determine the point of intersection (4,5)(1,3)(5,4)(3,1)

Answers

The intersection point is (4,5) because that's the point where the two lines meet

Using only a compass and straightedge, construct an isosceles triangle with base and legs congruent to the segments shown below.

Answers

I will write out the procedure to draw the isosceles triangle;

- Using the straight edge, measure the length of the base and draw a straight line congruent to the base length.

- Measure the length of the legs.

- Using the compass, set the compass to the same length as the length of the legs.

- From each edge of the base line drawn, Use the compass (already set to the length of the legs) to draw an arc each at the top of the base line;

- Then join each edge of the base line to the point at which the two arcs meet.

- Then you'll have an isosceles triangle, with base and legs as shown above.

Complete the table. Write each exponent as a simplified fraction or integer.
6 to the power of - 1
6 to the power of - 2
6 to the power of - 3
6 to the power of - 4

Answers

Answer + Explanation:

Since you just flip the fraction if you have a negative power, you get

1.) 6 to the power of -1 = 1/6

2.) 6 to the power of -2 = 1/36

3.) 6 to the power of -3 = 1/216

4.) 6 to the power of -4 = 1/1296

The height of a triangle is 5 less than double its base, which is 13/2. Find the triangle´s area.

Answers

Answer:

The answer you are looking for would be 11 7/2.

Step-by-step explanation:

You can find more information about this question here at:
https://brainly.in/question/41169338

May I have Brainliest please? My next rank will be the highest one: A GENIUS! Please help me on this journey to become top of the ranks! I would really appreciate it, and it would make my day! Thank you so much, and have a wonderful rest of your day!

Sara needs to purchase one notebook and packs of pencils (p) for school. A notebook costs $12.99 and packs of pencils cost $1.19 each. Which equation shows the relationship between the total cost, C, of a notebook and p packs of pencils?

Answers

The total cost of all the items mentioned can be expressed as

12.99 + 1.19 P = C

Given : Sara needs to purchase one notebook

Also it is given that pack of pencils is represented by P

Since a notebook costs = $ 12.99

And pack of pencils cost = $ 1.19

Thus the total cost can simply be represented by adding all the above mentioned items

this means it can be written as

12.99 + 1.19 P = C where C represents the total cost { given }

To know more about linear equation or related problems you may visit the link which is mentioned below;

https://brainly.com/question/13738061

#SPJ1

2v²+8v=-7 i need 5 points to graph range solutions

Answers

Given the quadratic equation below

[tex]\begin{gathered} 2v^2+8v=-7 \\ \Rightarrow2v^2+8v+7=0 \\ \therefore y=2v^2+8v+7 \end{gathered}[/tex]

Note: y is function defined by v ( independent variable)

To plot the graph of the function above, we will take a range of values for v, i.e

[tex]\begin{gathered} -2\le v\le2 \\ \text{The range}\Rightarrow-2,-1,0,1,2 \end{gathered}[/tex]

Given the range of values above, find the corresponding y-values . This can be done by substituting for v in the quadratic equation above

[tex]\begin{gathered} \text{when v=-2} \\ y=2(-2)^2+8(-2)+7=2(4)-16+7=8+7-16=15-16=-1 \end{gathered}[/tex][tex]\begin{gathered} \text{When v=-1} \\ y=2(-1)^2+8(-1)+7=2(1)-8+7=2+7-8=9-8=1 \end{gathered}[/tex][tex]\begin{gathered} \text{When v=0} \\ y=2(0)^2+8(0)+7=0+0+7=7 \end{gathered}[/tex][tex]\begin{gathered} \text{When v=1} \\ y=2(1)^2+8(1)+7=2+8+7=17 \end{gathered}[/tex][tex]\begin{gathered} \text{When v=2} \\ y=2(2)^2+8(2)+7=2(4)+16+7=8+16+7=31 \end{gathered}[/tex]

The table of values is shown below

Hence, the graph of the solution is shown below

1) When full, a tank contains 1620 gallons of water. Currently, the tank has 648 gallons of water in it. What percent of the full tank is this?​

Answers

Answer:

The tank is currently 40% (40 percent) full.

Step-by-step explanation:

We can find this by dividing the current fullness (648 gal) by the full tank (1620 gal).

648 / 1620 = 0.4

0.4 = 40%

How do we know to divide 648 by 1620 and not the other way around?

We know this because that we want to know what percent of the full tank is currently full. It's something that you just need to get used to, once you do enough of these types of problems, I'm sure you'll get the hang of it!

Hope I could help you! If I did, it would mean a lot of you could give braniest or even just a thanks :) If I didn't quite answer your question, please let me know and I'll try my best to assist you!

help help help help help help help help help help help help help help help help help help helppppooppppppppp

Answers

From the given figure

The prism formed from 5 cubes long, 2 cubes wide, and 2 cubes height, then

Its length = 5 cubes

Its width = 2 cubes

Its height = 2 cubes

Since the side of each cube = 1/2 cm, then

Its length = 5 x 1/2 = 5/2 cm

Its width = 2 x 1/2 = 1 cm

Its height = 2 x 1/2 = 1 cm

The volume of the prism = length x width x height, then

[tex]\begin{gathered} V=\frac{5}{2}\times1\times1 \\ V=\frac{5}{2}cm^3 \end{gathered}[/tex]

The volume of the prism is 5/2 cubic cm

The answer is A

4.
Jada drinks a cup of tea every morning at 8:00 am. for 14 days. There is 40 mg of
caffeine in each cup of tea she drinks. 24 hours after she drinks the tea, only 6% of
the caffeine is still in her body.
How much caffeine is in her body right after drinking the tea on the first, second, and third day?
1st day:
2nd day:
3rd day:
mg.
mg.
mg.

Answers

The amount of caffeine in her body on the 1st day is, 2.4 mg

The amount of caffeine in her body on the 2nd day is, 4.8 mg

The amount of caffeine in her body on the 3rd day is, 7.2 mg

Given, Jada drinks a cup of tea every morning at 8:00 am. for 14 days. There is 40 mg of caffeine in each cup of tea she drinks. 24 hours after she drinks the tea, only 6% of the caffeine is still in her body.

So, the amount of caffeine in her body on the 1st day is,

40×6/100

24/10

= 2.4 mg caffeine

The amount of caffeine in her body on the 2nd day is,

= 2× 2.4

= 4.8 mg caffeine

The amount of caffeine in her body on the 3rd day is,

= 3× 2.4

= 7.2 mg caffeine

Hence, The amount of caffeine in her body on the 1st day is, 2.4 mg

The amount of caffeine in her body on the 2nd day is, 4.8 mg

The amount of caffeine in her body on the 3rd day is, 7.2 mg

Learn more about Unitary Method here https://brainly.com/question/28627316

#SPJ9

Brandon buys a two-quart bottle of juice for $8.32. What is the unit rate of the cost of the juice per fluid ounce? 1 gallon = 1 gallon= 4 quarts 4 quarts 1 quart = 1 quart= 2 pints 2 pints 1 pint = 1 pint= 2 cups 2 cups 1 cup = 1 cup= 8 fluid ounces 8 fluid ounces Before you try that problem, answer the question below. How many fluid ounces of juice did Brandon buy? 64 64 What is the cost of juice in dollars per fluid ounce?

Answers

The cost of the juice in dollars per fluid ounce s $0.13..

What is the cost of the juice?

The first step is to convert quart to ounces.

1 quart = 32 ounces

The quantity of juice bought in fluid ounces : quantity in quart x unit of conversion=  32 x 2 = 64 fluid ounces

The next step is to divide the cost of the bottle of juice by the converted capacity of the juice bottle. Division is the mathematical process of grouping a number into equal parts using another number.

The cost of the juice in dollars per fluid ounce = cost of the bottle / capacity of the bottle in fluid ounces

= $8.32 / 64 = $0.13

To learn more about division, please check: https://brainly.com/question/13281206

#SPJ1

Brandon has a jar of quarters and dimes with a total value of $6.05. The number of quarters is 7 less than twice the number of dimes. How many quarters and how many dimesdoes Brandon have?Number of quarters -Number of dimes -

Answers

SOLUTION:

In this problem.

Let number of dimes = x, then number of quarters = 2x - 7.

The monetary values of quarters and dimes are;

[tex]\begin{gathered} 0.1x+0.25(2x-7)=6.05 \\ 0.1x+0.5x-1.75=6.05 \\ 0.6x=7.8 \\ x=13 \end{gathered}[/tex]

Thus, 2( 13) - 7 = 19.

Therefore, there are 13 dimes and 19 quarters.

(d) In 7 years Bernard will be twice as old as he was 8 years ago. How old is Bernard now?​

Answers

The present age of the Bernard is, 23.

What is age?

Given a date of birth, this free age calculator calculates age in terms of years, months, weeks, days, hours, minutes, and seconds.

Calculated using the population's arithmetic mean, average age. The median age is a factor that determines the population's average age.

As given, in 7 years Bernard will be twice as old as he was 8 years ago.

Let  the present age of Bernard = x

Bernard age after 7 years = x + 7.

Bernard age before 8 years = x - 8.

From the given information,

2(x - 8) = x + 7

2x - 16 = x + 7

2x - x = 16 + 7

x = 23

Therefore, the present age of Bernard is 23.

To know more about the age, click on the link

https://brainly.com/question/343671

#SPJ1

Bryce wants to renovate his kitchen. The construction estimate is $28,400. He can finance through his local bank or directly through the construction company. The bank (loan A) offers a maximum 6-year loan at 8% annual interest, resulting in monthly payments of $497.94.The construction company (loan B) offers a maximum 10-year loan at 5.5% annual interest, resulting in monthly payments of $308.21.Which loan requires Bryce to pay less, and by how much?First, calculate the total payback for each loan.

Answers

ANSWER:

Loan A requires Bryce to pay less by $1,133.52

EXPLANATION:

For bank A, we're given;

Length of loan in years(t) = 6 years

Annual interest rate(r) = 8% = 8/100 = 0.08

Loan payment(PMT) = $497.94

Number of compounds per year(n) = 12

We can go ahead and determine the loan amount using the below formula as seen below;

[tex]\begin{gathered} Loan\text{ }amount,P_0=\frac{PMT(1-(1+\frac{r}{n})^{-nt})}{\frac{r}{n}} \\ \\ =\frac{497.94(1-(1+\frac{0.08}{12})^{-12*6}}{\frac{0.08}{12}} \\ \\ =\text{ \$}28399.77 \end{gathered}[/tex]

So we can see that Bryce will pay back;

[tex]\begin{gathered} Bank\text{ }A\text{ }Loan\text{ }Payback=497.94(12*6) \\ \\ =497.94\left(72\right) \\ \\ =\text{\$}35,851.68 \end{gathered}[/tex]

For bank B, we're given;

Length of loan in years(t) = 10 years

Annual interest rate(r) = 5.5% = 5.5/100 = 0.055

Loan payment(PMT) = $308.21

Number of compounds per year(n) = 12

We can go ahead and determine the loan amount using the below formula as seen below;

[tex]\begin{gathered} Loan\text{ }amount,P_0=\frac{PMT(1-(1+\frac{r}{n})^{-nt})}{\frac{r}{n}} \\ \\ =\frac{308.21(1-(1+\frac{0.055}{12})^{-12*10}}{\frac{0.055}{12}} \\ \\ =\text{ \$}28399.57 \end{gathered}[/tex]

So we can see that Bryce will pay back;

[tex]\begin{gathered} Bank\text{ B }Loan\text{ }Payback=308.21(12*10) \\ \\ =308.21\left(120\right) \\ \\ =\text{\$}36985.20 \end{gathered}[/tex]

We can see that Bryce will pay back $35,851.68 for Loan A and $36,985.20 for Loan B, so Loan A requires Bryce to pay less and by $1,133.52

In a class of 30 students, 3 of the 16 boys are left-handed and 2 of the girls are left-handed. One student is chosen at random . Which of the following represents a conditional probability ?

Answers

The conditional probability is represented in option A : the probability that the student is left-handed, given that the student is a boy.

The total number of students in the class is 30. The number of boys in the class is 16. The number of boys who are left-handed is 3. The number of left-handed girls is 2. One student is chosen randomly. We need to find a situation that represents a conditional probability. The conditional probability occurs when the probability of one event depends on the occurrence of another event. So, the situation suitable for this to happen is to find the probability that the student is left-handed, given that the student is a boy.

The complete question is :

In a class of 30 students, 3 of the 16 boys are left-handed, and 2 of the 14 girls are left-handed. One student is chosen at random. Which of the following represents a conditional probability?

A. The probability that the student is left-handed, given that the student is a boy

B. The probability that the student is a girl and left-handed

C. The probability that the student is right-handed

D. The probability that the student is not a boy

To learn more about probability, visit :

https://brainly.com/question/11234923

#SPJ9

what does pie mean in math

Answers

Answer: The symbol used by mathematicians to represent the ratio of a circle's

Step-by-step explanation: circumference to its diameter is the lowercase Greek letter π, sometimes spelled out as pi. In English, π is pronounced as "pie"

Jerome is making beef stew for his family. The recipe calls for 3.5 pounds (lbs.) of stew meat. He notices that he has 1.25 pounds (lbs.) in his refrigerator. How much stew meat should buy when he goes to the grocery in order to have enough? a) What operation (add, subtract, multiply or divide) would you use to solve this? b) How did you know?

Answers

EXPLANATION

Let's see the facts:

Call for---> 3.5 lbs of stew meat

Refrigerator -------> 1.25 lbs stew meat

He would buy:

Required - Available ----> 3.5 - 1.25 = 2.25 lbs of stew meat

The operation that we apply is subtraction because 3.5lbs (required) is greater than 1.25lbs (available).

Taylor received a paycheck of $1575. First, she spent 30% of her paycheck to make a house payment. Next, she spent 3/14 of the remaining amount to make a car payment. What amount was still left over from Taylor's paycheck after these two payments?

Answers

$866.25

1) Gathering the data

$1575

she spent 30%

she spent 3/14

2) Let's calculate it, starting with the 30% expense. Writing a factor, we have already the value:

1575 x ( 1 -0.3)

1575 x (0.7)

1102.50

Now let's proceed with the deduction of 3/14, which can be calculated through that:

1102.5 -3/14x (1102.5)

$866.25

3) Hence, Taylor's balance was after those payments of $866.25

Order the numbers 7/4 , 1 .6, 1 5/8 , 1. 62 from least to greatest.

Answers

Answer:

1.6, 1.62, 1 5/8, 7/4

Step-by-step explanation:

First, let's turn the fractions into decimals:

For 7/4 you simply divide 7 by 4. Which equals 1.75.

Next for 1 5/8 we divide 5 by 8, which equals 0.625. + the 1 in 1 5/8. So we have 1.625.

Now, we arrange the decimals:

1.6, 1.62, 1.625, 1.75

But this is NOT the answer. We have to replace the decimals that WERE fractions back to the original, then order them from there:

1.6, 1.62, 1 5/8, 7/4.

This IS the answer.

May I have Brainliest please? My next rank will be the highest one: A GENIUS! Please help me on this journey to become top of the ranks! I would really appreciate it, and it would make my day! Thank you so much, and have a wonderful rest of your day!

Answer:

the answer to this question is in the screenshot. pls mark this answer as the brainliest

Step-by-step explanation:

Which of the following isn’t a solution to cos 2x + cos x=0 on the interval [0,2pi)

Answers

Given:

There are given the equation:

[tex]cos2x+cosx=0[/tex]

Explanation:

According to the question:

We need to find the value where the given equation is satisfied.

So,

From the equation:

Put the 0 for x for the option first.

[tex]\begin{gathered} cos2x+cosx=0 \\ cos2(0)+cos(0)=0 \\ 1+1=0 \\ 2\ne0 \end{gathered}[/tex]

Then,

For the second option:

[tex]\begin{gathered} cos2x+cosx=0 \\ cos2(\frac{\pi}{3})+cos\frac{\pi}{3}\ne0 \end{gathered}[/tex]

For option third:

[tex]\begin{gathered} cos2x+cosx=0 \\ cos2(\pi)+cos(\pi)=0 \\ -1+2cos^2(\pi)+cos(\pi)=0 \\ -1+2(-1)^2-1=0 \\ -1+2-1=0 \\ 0=0 \end{gathered}[/tex]

Final answer:

Hence, the correct option C.

The random variable x has a normal distribution with standard deviation. It is known that the probability that x exceeds is. 90. Find the mean of the probability distribution.

Answers

The random variable x has a normal distribution with standard deviation. It is known that the probability that x exceeds is. 90. Therefore, mean  of the probability distribution is 133.09.

We are given that the random variable x has a normal distribution with standard deviation 21,i.e;

X ~ N( u, σ=21)

The Z probability is given by;

             Z = X- u/σ ~ (0,1)

Also, it is known that the probability that x exceeds 160 is 0.90 ,i.e;

P(X > 160) = 0.90

P( X-u/σ > 160-u /21  ) = 0.90

From the z% table we find that at value of x = -1.2816, the value of

P(X > 160) is 90%

which means;   160-u /21   = -1.2816

                          160 -  = 21*(-1.2816)

                             = 160 - 26.914 = 133.09

Therefore, mean  of the probability distribution is 133.09.

learn more about of mean here

https://brainly.com/question/15028000

#SPJ4

                         

1. What is the equation of the line in standard form? A302-10

Answers

Question: What is the equation of the line in standard form?

The standard form for a line is :

y = mx + b

where m is the slope and b is the intercept with y axis.

First, we are going to find the slope, choosing any two points on the line. For example (X1, Y1) = (-4,0) and (X2, Y2) = (0,3). Then, by definition the slope is:

[tex]m\text{ = }\frac{Y2\text{ - Y1}}{X2\text{ - X1}}\text{ = }\frac{3-0}{0-(-4)}\text{ = }\frac{3}{0+\text{ 4}}\text{ = }\frac{3}{4}[/tex]

so, our new line equation would be:

[tex]y\text{ = mx + b = }\frac{3}{4}x\text{ + b}[/tex]

that is:

[tex]y\text{ = }\frac{3}{4}x\text{ + b}[/tex]

Now, we are going to find the y-intercept. This is also accomplished by picking two points on the line and solving for b. For example (X2, Y2) = (0,3). So, for above equation we have:

[tex]3\text{ = }\frac{3}{4}(0)\text{ + b}[/tex]

then

[tex]3\text{ = 0 + b = b}[/tex]

Then, we have b = 3.

Now, replacing the values of the slope and the intercept previously found, we obtain the equation of the line :

[tex]y\text{ = mx + b = }\frac{3}{4}x\text{ + }3[/tex]

that is

[tex]y\text{ = }\frac{3}{4}x\text{ + }3[/tex]

What is 4 1/4 - 2 1/2

Answers

1.75
idrk i searched it up

Answer:

=1 and 3 over 41

=1.75

Step-by-step explanation:

Which order pair can be plotted together with these four points, so that the resulting graph still represents a function.

Answers

The ordered pair that can be plotted together with these four points is

(0, 3).

Option C is the correct answer.

What is a function?

function is a relationship between inputs where each input is related to exactly one output.

Example:

f(x) = 2x + 1

f(1) = 2 + 1 = 3

f(2) = 2 x 2 + 1 = 4 + 1 = 5

The outputs of the functions are 3 and 5

The inputs of the function are 1 and 2.

We have,

The following coordinates are from the graph.

(3, -1), (2, 3), ), (-4, 2), and (-2, -3)

We know that,

A function can have many-to-one but not one-to-many.

So,

We can not have (-2, 0), (-2, 3). and (3, 0) because we already have (-2, 3) and (3, -1).

This is an indication of a one-to-many function that is not possible.

(0, 3) is possible.

Thus,

The ordered pair that can be plotted together with these four points is

(0, 3).

Option C is the correct answer.

Learn more about functions here:

https://brainly.com/question/28533782

#SPJ1

50% of the tickets sold an amusement park or a discount tickets at the park sold 94 tickets to know how many discount tickets did it sell

Answers

we have that

50%=50/100=0.50

Multiply the total tickets sold by the factor of 0.50

so

94*0.50=47 tickets

the answer is 47 tickets

a semi-trailer is being packed with boxes that are 1 and 1/2 feet on each Edge the cargo space of the trailer is 28 ft long eight and a half feet wide and 9 and 1 over 6 feet high what is the maximum number of boxes that will fit inside the trailer

Answers

The maximum number of boxes that will fit inside the trailer according to the available volume is 540.

A semi-trailer is being packed with boxes. The side length of each box is 1.5 feet. The cargo space of the trailer is 28 feet in length, 8.5 feet wide and 9.167 feet high. Let the number of boxes that can fit according to the length, width, and height of the trailer be x, y, and z. The values of x, y, and z are the greatest whole numbers less than or equal to the ratio of the length, width, and height of the trailer to the length, width, and height of the box, respectively. The first inequality is x ≤ 28/1.5 ≤ 18.67, so the value of x is 18. The second inequality is y ≤ 8.5/1.5 ≤ 5.67, so the value of y is 5. The third inequality is z ≤ 9.167/1.5 ≤ 6.11, so the value of z is 6. The total number of boxes is the product of the three variables.

P = x*y*z = 18*5*6 = 540

To learn more about volume, visit :

brainly.com/question/1578538

#SPJ9

Other Questions
I just need to know the answer for question 12 Solve the puzzleABCDEFGHIJKLMNOP1101134NUCLEAR _ A ____L6 19 10 42 26 3 249 4320 2613 3E4 13 53LUN AR L.4 1F Athineon 2020ALL-4 4 123 20 4 19 6 15 3 2920 7 20 26NE1320 66BQR20 2E9 13 16ARRE24 3 2 2 9 13 7N(__.10 22 20 20 16 9R24 20 2 13STUVWXYZ191E10 20 24 25 20 16 13 7LE10 20 19 25 4 13 1665AREN25 3 2 13 6 15 22 2010 22 9L EN.4 7 2 13 6led maths &simalein subjectof teacherderCholeping inor roleel andsponson help to raisecontrol, Privateof local authorityAcademies-Taken outthat are runreligious ethosT X Por Han spent a total of $221.76, before tax, on bags of chips for the basketball team, and each bag cost $3.52. What is the total number of bags of chips that Han bought? - 16mn-(-25mn)+(-7mn)Which of the following is equivalent to the expression above? What were the primary causes of the French Revolution? select four options.1.lack of representation for the lower classes.2.the establishment of a state religion. 3.oppressive leadership.4.high taxes on the lower class.5.high rates of poverty. The product of the ages (in days) of two newborn babies Simran and Jessie in two dayswill be 48 more than the product of their ages today. How old are the babies if Jessie is 2days older than Simran? The probability that VSU and KSU both win a basketball game in the same week is 47%. Theprobability that VSU wins is 50%. What is the probability that KSU will win given that VSUhas already won? Simple calculator test - determine the angle, , to the nearest whole degree from each of the following:A. cos() = 0.2079B. tan() = 0.2126C. sin() = 0.5446 Consider the quadratic function.What are the x-intercepts and y-intercept?What is the equation of the axis of symmetry?What are the coordinates of the vertex?Graph the function on the coordinate plane. Include the axis of symmetry. What are the advantages and disadvantages of public transit?Give me one advantage or disadvantage of public transit and use the template below to give me 2-4 sentences?Supporting Paragraph #1 Topic Sentence (The main point of your first support paragraph):____________________________________________________________________________ Which values has three significant figures?14001001101.22310 An equilibrium mixture, at 529C in a 1208-mL container, involving the chemical systemN2(g) + 3H2(g) == 2NH3(g)is found to contain 3.62 g of N2, 0.773 g of H2, and 0.498 g of NH3. Calculate the equilibrium constant (Keq expressed in terms of the molar concentrations) at the given temperature. the sum of 2 numbers is 75. the second number is 3 less than twice the first. find the numbers what is your stance on the subject physical education being undervalued what is the value of x would make lines l and m parrallel How would you prepare 250 mL of 0.125 M HCl from concentrated HCl (aq) that is 38.0% by mass with a density of 1.19 g/mLnote: find moles in 250 mL of 0.125 HCl whats the variance and the standard deviation of the data set 6,9,2,5,4,10,3,12,2,7,7,8 What are the coordinates of point P on the directed line segment from R to Q such that 5 Pis 6 the length of the line segment from R to Q? Round to the nearest tenth, if necessary. A (-3.5, 2.3 ) B. (3.5, -2.3 ) C. (-3, 2) D. (3, 2) The ratio of the length of an airplane wing to its width is 9 to 1. If the length of a wing is 43.9 metrs, how wide must it be? The airplane wing must be ____ meters wide. (Round to the nearest hundredth.) Consider the equation 2x+5y=1Identify the slope and y intercept